\( \newcommand{\N}{\mathbb{N}} \newcommand{\R}{\mathbb{R}} \newcommand{\C}{\mathbb{C}} \newcommand{\Q}{\mathbb{Q}} \newcommand{\Z}{\mathbb{Z}} \newcommand{\P}{\mathcal P} \newcommand{\B}{\mathcal B} \newcommand{\F}{\mathbb{F}} \newcommand{\E}{\mathcal E} \newcommand{\brac}[1]{\left(#1\right)} \newcommand{\abs}[1]{\left|#1\right|} \newcommand{\matrixx}[1]{\begin{bmatrix}#1\end {bmatrix}} \newcommand{\vmatrixx}[1]{\begin{vmatrix} #1\end{vmatrix}} \newcommand{\lims}{\mathop{\overline{\lim}}} \newcommand{\limi}{\mathop{\underline{\lim}}} \newcommand{\limn}{\lim_{n\to\infty}} \newcommand{\limsn}{\lims_{n\to\infty}} \newcommand{\limin}{\limi_{n\to\infty}} \newcommand{\nul}{\mathop{\mathrm{Nul}}} \newcommand{\col}{\mathop{\mathrm{Col}}} \newcommand{\rank}{\mathop{\mathrm{Rank}}} \newcommand{\dis}{\displaystyle} \newcommand{\spann}{\mathop{\mathrm{span}}} \newcommand{\range}{\mathop{\mathrm{range}}} \newcommand{\inner}[1]{\langle #1 \rangle} \newcommand{\innerr}[1]{\left\langle #1 \right \rangle} \newcommand{\ol}[1]{\overline{#1}} \newcommand{\toto}{\rightrightarrows} \newcommand{\upto}{\nearrow} \newcommand{\downto}{\searrow} \newcommand{\qed}{\quad \blacksquare} \newcommand{\tr}{\mathop{\mathrm{tr}}} \newcommand{\bm}{\boldsymbol} \newcommand{\cupp}{\bigcup} \newcommand{\capp}{\bigcap} \newcommand{\sqcupp}{\bigsqcup} \newcommand{\re}{\mathop{\mathrm{Re}}} \newcommand{\im}{\mathop{\mathrm{Im}}} \newcommand{\comma}{\text{,}} \newcommand{\foot}{\text{。}} \)

Tuesday, May 25, 2010

不等式小記

Problem (Austrian Mathematical Olympiad 2008)Prove that the inequality $ \displaystyle \sqrt{a^{1-a}b^{1-b}c^{1-c}}\leq \frac{1}{3}$ holds for all positive real numbers $  a,b,c$ with $  a+b+c=1$.

Those who always discuss math with me will know how to solve it, that's not hard. Now it can be easily generalized to \[ \sqrt{a_1^{1-a_1}a_2^{1-a_2}\cdots a_n^{1-a_n}}\leq \frac{1}{n^{(n-1)/2}}\] with $  a_1+a_2+\cdots +a_n=1$. From this we get for any $  a_i>0$,\[\frac{a_1+a_2+\cdots+a_n}{n}\ge \left(\frac{a_1a_2\cdots a_n}{(a_1^{a_1}a_2^{a_2}\cdots a_n^{a_n})^{1/\sum_{i=1}^na_i}}\right)^{1/(n-1)}.\]
We have established an extremely ugly lower bound. This inequality is useful if we are given a condition on $  \prod a_i^{a_i}$, for example from now on I can show that if \[\boxed{x^xy^yz^z=1, x,y,z>0},\] then \[ \sum_{cyc}\sqrt{\frac{x}{yz}}\ge 3.\]
Take $n =2$, set $  (a_1,a_2)=(x,y)$. The inequality reduces to \[\tfrac{1}{2}(x+y)\ge (x^yy^x)^{1/(x+y)}\iff \left(\tfrac{1}{2}(x+y)\right)^{x+y}\ge x^yy^x
\] (at least I think this inequality is somehow useful), by a little bit argument the following holds for all positive $ x,y$, \[
\sqrt[m]{\frac{x^m+y^m}{2}}\mathop{\ge}\limits_{m\ge n}\sqrt[n]{\frac{x^n+y^n}{2}}\mathop{\ge}\limits_{n\in\mathbb{N}}\frac{x+y}{2}\ge \sqrt{xy}\ge \frac{2}{\frac{1}{x}+\frac{1}{y}}\ge (x^yy^x)^{1/(x+y)}.\]

No comments:

Post a Comment